Question

Three dimensions. Three point particles are fixed in place in an xyz coordinate system. Particle A,...

Three dimensions. Three point particles are fixed in place in an xyz coordinate system. Particle A, at the origin, has mass mA. Particle B, at xyz coordinates (3.00d, 2.00d, 4.00d), has mass 3.00mA, and particle C, at coordinates (–3.00d, 3.00d, –2.00d), has mass 3.00mA. A fourth particle D, with mass 3.00mA, is to be placed near the other particles. If distance d = 9.90 m, at what (a) x, (b) y, and (c) z coordinate should D be placed so that the net gravitational force on A from B, C, and D is zero?

I need the answers in meter please and many thanks.

Homework Answers

Know the answer?
Your Answer:

Post as a guest

Your Name:

What's your source?

Earn Coins

Coins can be redeemed for fabulous gifts.

Not the answer you're looking for?
Ask your own homework help question
Similar Questions
One dimension. In the figure, two point particles are fixed on an x axis separated by...
One dimension. In the figure, two point particles are fixed on an x axis separated by distance d. Particle A has mass mA and particle B has mass 6.00 mA. A third particle C, of mass 73.0 mA, is to be placed on the x axis and near particles A and B. In terms of distance d, at what x coordinate should C be placed so that the net gravitational force on particle A from particles B and C is...
Three particles are fixed in positions on the xy plane. Particle A, with a mass of...
Three particles are fixed in positions on the xy plane. Particle A, with a mass of 6.00 g, is located at the origin. Particle B, with a mass of 12.0 g, is located in the second quadrant a distance 0.500 m from the origin at an angle of 30◦ from the x axis. Particle C has a mass of 8.00 g. The net gravitational force acting on particle A due to particles B and C is 2.77 × 10−14 N...
A point charge is place at the origin of a cartesian coordinate system. Another point charge...
A point charge is place at the origin of a cartesian coordinate system. Another point charge is placed at the coordinates (0, -5.0 m), and a third charge is placed at (7.0 m, 0). If all three particles have a charge of 3.0 C, find the direction (measured in degrees from the negative x-axis) of the net force on the charge at (0, -5.0 m).
A point charge is place at the origin of a cartesian coordinate system. Another point charge...
A point charge is place at the origin of a cartesian coordinate system. Another point charge is placed at the coordinates (0, -5.0 m), and a third charge is placed at (7.0 m, 0). If all three particles have a charge of 3.0 C, find the magnitude of the net force on the charge at (0, -5.0 m) Please explain the steps in detail for me and write legibly. Thank you.
Point particles A and B are fixed in place on the y axis. particle A has...
Point particles A and B are fixed in place on the y axis. particle A has a charge of QA=59 microC and is located a distance d=2.96 cm above the x axis. Particle B has a charge of -QB and is located a distance 2.96 cm below the x axis. Particle QC = 12.6 microC and is relaeased at rest on the positive x axis a distance D=7.68 cm from the y axis. The initial acceleration of particle C is...
Three particles are placed in the xy plane. A m1 = 65 g particle is located...
Three particles are placed in the xy plane. A m1 = 65 g particle is located at (x1, y1), where x1 = 4.4 m and y1 = 1.7 m and a m2 = 58 g particle is located at (x2, y2), where x2 = −1.4 m and y2 = −6.6 m . A) What must be the x coordinate of the m3 = 42 g particle so that the center of mass of the three-particle system is at the origin?...
Consider a vertical plane in a constant gravitational field. Let the origin of a coordinate system...
Consider a vertical plane in a constant gravitational field. Let the origin of a coordinate system be located at some point in this plane. A particle of mass m moves in the vertical plane under the influence of gravity and under the influence of an additional force f=-Ara-1 directed toward the origin (r is the distance from the origin and A and a are constants). Choose appropriate generalized coordinates and find the Lagrangian equations of motion.
In the figure particles 1 and 2 are fixed in place on an x axis, at...
In the figure particles 1 and 2 are fixed in place on an x axis, at a separation of L = 6.50 cm. Their charges are q1 = +e and q2 = -64e. Particle 3 with charge q3 = +5e is to be placed on the line between particles 1 and 2, so that they produce a net electrostatic force F3 net on it. (a) At what coordinate should particle 3 be placed to minimize the magnitude of that force?...
The drawing shows three particles far away from any other objects and located on a straight...
The drawing shows three particles far away from any other objects and located on a straight line. The masses of these particles are mA = 324 kg, mB = 527 kg, and mC= 183 kg. Take the positive direction to be to the right. Find the net gravitational force, including sign, acting on (a) particle A, (b) particle B, and (c) particle C.
Three particles sit on the vertices of an equilateral triangle with sides of d = 5.00...
Three particles sit on the vertices of an equilateral triangle with sides of d = 5.00 cm. Particle 1 has a mass of 25.0 kg, particle 2 has a mass of 20.0 kg, and particle 3 has a mass of 30.0 kg. What is the direction of the net gravitational force on mass 1?
ADVERTISEMENT
Need Online Homework Help?

Get Answers For Free
Most questions answered within 1 hours.

Ask a Question
ADVERTISEMENT
Active Questions
  • Movie stars and U.S. presidents have fished Pyramid Lake. It is one of the best places...
    asked 9 minutes ago
  • what advantages do percent distributions have over frequency distributions when comparing populations at different years?
    asked 15 minutes ago
  • Richard has just been given a 6-question multiple-choice quiz in his history class. Each question has...
    asked 36 minutes ago
  • Write about westernized elements in life; westernization in our life, and this includes Americanization.
    asked 1 hour ago
  • The college Physical Education Department offered an Advanced First Aid course last summer. The scores on...
    asked 1 hour ago
  • Researchers hypothesized that increasing a woman's level of arousal would increase her perceptions of attractiveness of...
    asked 2 hours ago
  • Equation 37-14b in the textbook gives the energy emitted by Hydrogen when electrons transition between states...
    asked 2 hours ago
  • Theory of Computation Please provide explanation too on how it works a. Give an NFA recognizing...
    asked 2 hours ago
  • Question 1 A sequential pattern detection circuit (state machine) has input A and output Y, which...
    asked 2 hours ago
  • Natural Foods Inc. is planning to invest in new manufacturing equipment to make a new garden...
    asked 2 hours ago
  • Explain why a callable bond's price would be expected to decline less than an otherwise comparable...
    asked 2 hours ago
  • . What are the three types of pricing strategies services that services employ? Using a real...
    asked 3 hours ago